2018年7月20日 星期五

九十六學年度數學學科能力測驗

大學入學考試中心
九十六學年度學科能力測驗試題

數學考科



-作答注意事項-

  1. 考試時間:$100$分鐘
  2. 題型題數:單選題$5$題,多選題$6$題,選填題第$A$至$I$題共$9$題
  3. 作答方式:
    • 用2B鉛筆在「答案卡」上劃記,修正時應以橡皮擦拭,切勿使用修正液
    • 答錯不倒扣
  4. 作答說明:在答案卡適當位置選出數值或符號。請仔細閱讀下面的例子。
    1. 填答選擇題時,只用$1$,$2$,$3$,$4$,$5$等五個格子,而不需要用到$-$,$±$,以及$6$,$7$,$8$,$9$,$0$等格子。
      例:若第$1$題的選項為(1)$3$ (2)$5$ (3)$7$ (4)$9$ (5)$11$,而正確的答案為$7$,亦即選項(3)時,考生要在答案卡第$1$列的$\underset{\boxed{~~}}{3}$劃記(注意不是$7$)如:

      $\begin{array}{|c|}\hline解答欄\\\hline1~~\underset{\boxed{~~}}{-}~\underset{\boxed{~~}}{±}~\underset{\boxed{~~}}{1}~\underset{\boxed{~~}}{2}~\underset{\color{black}{▆▆}}{3}~\underset{\boxed{~~}}{4}~\underset{\boxed{~~}}{5}~\underset{\boxed{~~}}{6}~\underset{\boxed{~~}}{7}~\underset{\boxed{~~}}{8}~\underset{\boxed{~~}}{9}~\underset{\boxed{~~}}{0}~\end{array}$

      例:若多選題第$10$題的正確選項為(1)與(3)時,考生要在答案卡的第$10$列的$\underset{\boxed{~~}}{1}$與$\underset{\boxed{~~}}{3}$劃記,如:

      $\begin{array}{|c|}10~~\underset{\boxed{~~}}{-}~\underset{\boxed{~~}}{±}~\underset{\color{black}{▆▆}}{1}~\underset{\boxed{~~}}{2}~\underset{\color{black}{▆▆}}{3}~\underset{\boxed{~~}}{4}~\underset{\boxed{~~}}{5}~\underset{\boxed{~~}}{6}~\underset{\boxed{~~}}{7}~\underset{\boxed{~~}}{8}~\underset{\boxed{~~}}{9}~\underset{\boxed{~~}}{0}\\\hline\end{array}$

    2. 選填題的題號是A,B,C,……,而答案的格式每題可能不同,考生必須依各題的格式填答,且每一個列號只能在一個格子劃記。

      例:若第B題的答案格式是$\displaystyle\underline{\frac{⑱}{⑲}}$,而依題意計算出來的答案是$\displaystyle\frac{3}{8}$,則考生必須分別在答案卡上的第$18$列的$\underset{\boxed{~~}}{3}$與第$19$列的$\underset{\boxed{~~}}{8}$劃記,如:

      $\begin{array}{|c|}18~~\underset{\boxed{~~}}{-}~\underset{\boxed{~~}}{±}~\underset{\boxed{~~}}{1}~\underset{\boxed{~~}}{2}~\underset{\color{black}{▆▆}}{3}~\underset{\boxed{~~}}{4}~\underset{\boxed{~~}}{5}~\underset{\boxed{~~}}{6}~\underset{\boxed{~~}}{7}~\underset{\boxed{~~}}{8}~\underset{\boxed{~~}}{9}~\underset{\boxed{~~}}{0}\\19~~\underset{\boxed{~~}}{-}~\underset{\boxed{~~}}{±}~\underset{\boxed{~~}}{1}~\underset{\boxed{~~}}{2}~\underset{\boxed{~~}}{3}~\underset{\boxed{~~}}{4}~\underset{\boxed{~~}}{5}~\underset{\boxed{~~}}{6}~\underset{\boxed{~~}}{7}~\underset{\color{black}{▆▆}}{8}~\underset{\boxed{~~}}{9}~\underset{\boxed{~~}}{0}\end{array}$

      例:若第C題的答案格式是$\displaystyle\underline{\frac{⑳㉑}{50}}$,而答案是$\displaystyle\frac{-7}{50}$時,則考生必須分別在答案卡的第$20$列的$\underset{\boxed{~~}}{-}$與第$21$列的$\underset{\boxed{~~}}{7}$劃記,如:

      $\begin{array}{|c|}20~~\underset{\color{black}{▆▆}}{-}~\underset{\boxed{~~}}{±}~\underset{\boxed{~~}}{1}~\underset{\boxed{~~}}{2}~\underset{\boxed{~~}}{3}~\underset{\boxed{~~}}{4}~\underset{\boxed{~~}}{5}~\underset{\boxed{~~}}{6}~\underset{\boxed{~~}}{7}~\underset{\boxed{~~}}{8}~\underset{\boxed{~~}}{9}~\underset{\boxed{~~}}{0}\\\hline21~~\underset{\boxed{~~}}{-}~\underset{\boxed{~~}}{±}~\underset{\boxed{~~}}{1}~\underset{\boxed{~~}}{2}~\underset{\boxed{~~}}{3}~\underset{\boxed{~~}}{4}~\underset{\boxed{~~}}{5}~\underset{\boxed{~~}}{6}~\underset{\color{black}{▆▆}}{7}~\underset{\boxed{~~}}{8}~\underset{\boxed{~~}}{9}~\underset{\boxed{~~}}{0}\end{array}$

  5. ※試題後附有參考公式及可能用到的對數值與參考數值


第一部分:選擇題(佔$55$分)
  1. 單選題(佔$25$分)
  2. 說明:第$1$至$5$題,每題選出最適當的一個選項,劃記在答案卡之「解答欄」,每題答對得$5$分,答錯不倒扣。
    1. 設$f\left(x\right)=ax^6−bx^4+3x−2$,其中$a,b$為非零實數,則$f\left(5\right)-f\left(−5\right)$之值為
      1. $-30$
      2. $0$
      3. $2\sqrt{2}$
      4. $30$
      5. 無法確定(與$a,b$有關)
    2. 訣竅直接計算即可。
      解法代入$x=5$與$x=-5$後計算如下

      $f\left(5\right)-f\left(-5\right)=\left(a\cdot5^6-b\cdot5^4+3\cdot5-2\right)-\left(a\cdot\left(-5\right)^6-b\cdot\left(-5\right)^4+3\cdot\left(-5\right)-2\right)=15-\left(-15\right)=30$

      故選(4)。

    3. 試問共有多少個正整數$n$使得坐標平面上通過點$A\left(−n,0\right)$與點$B\left(0,2\right)$的直線亦通過點$P\left(7,k\right)$,其中$k$為某一正整數?
      1. $2$個
      2. $4$個
      3. $6$個
      4. $8$個
      5. 無窮多個
    4. 訣竅利用截距式寫出通過$A$與$B$的直線方程式,由於通過$P$點,代入後利用正整數的特性來判定可能的數量。
      解法利用截距式可得直線方程式為$\displaystyle\frac{x}{-n}+\frac{y}{2}=1$,接著代入$P$點坐標可得

      $\displaystyle\frac{7}{-n}+\frac{k}{2}=1$

      即有$14=n\left(k-2\right)$。當$n=1$時$k=16$;當$n=2$時$k=9$;當$n=7$時$k=4$;當$n=14$時$k=3$,故共有四個正整數$n$能使$k$為正整數,應選(2)。

    5. 設某沙漠地區某一段時間的溫度函數為$f\left(t\right)=−t^2+10t+11$,其中$1\leq t\leq10$,則這段時間內該地區的最大溫差為
      1. $9$
      2. $16$
      3. $20$
      4. $25$
      5. $36$
    6. 訣竅運用配方法來求極大值或極小值,並留意變數區間的影響。
      解法直接進行配方法可得$f\left(t\reight)=-\left(t-5\right)^2+36$,因此最大值為$f\left(5\right)=36$,而最小值為$f\left(10\right)=11$,故最大溫差為$36-11=25$,應選(4)。

    7. 坐標平面上方程式$\displaystyle\frac{x^2}{9}+\frac{y^2}{4}=1$的圖形與$\displaystyle\frac{\left(x+1\right)^2}{16}+\frac{y^2}{9}=1$的圖形共有幾個交點?
      1. $1$個
      2. $2$個
      3. $3$個
      4. $4$個
      5. $0$個
    8. 訣竅直接畫出橢圓的圖形尋找交點數即可;亦可直接解聯立方程。
      解法一直接繪圖如下
      如此可知恰有一個交點於$\left(3,0\right)$,故應選(1)。
      解法二對第一個方程式可以同乘以$4$為$\displaystyle\frac{4}{9}x^2+y^2=4$,而第二式可以同乘以$9$得$\displaystyle\frac{9}{16}\left(x+1\right)^2+y^2=9$。兩式相減可得

      $\displaystyle\frac{81\left(x+1\right)^2-64x^2}{144}=\frac{9}{16}\left(x+1\right)^2-\frac{4}{9}x^2=5$

      即有

      $17x^2+162x-639=0$

      可以直接因式分解得$\left(x-3\right)\left(17x+213\right)=0$,因此$x$有兩解。若$x=3$,則$y=0$;若$\displaystyle x=-\frac{213}{17}$,則$y$無解,故僅有一個交點。應選(1)。

    9. 關於坐標平面上函數$y=\sin x$的圖形和$\displaystyle y=\frac{x}{10\pi}$的圖形之交點個數,下列哪一個選項是正確的?
      1. 交點的個數是無窮多
      2. 交點的個數是奇數且大於$20$
      3. 交點的個數是奇數且小於$20$
      4. 交點的個數是偶數且大於或等於$20$
      5. 交點的個數是偶數且小於$20$
    10. 訣竅藉由圖形的對稱性與週期性進行觀察與分析。
      解法首先可以注意到$x=0$處會交會,再者由於兩者皆為奇函數,故會對稱於原點,從而會有奇數個交點。另一方面,由於正弦函數的值域為$\left[-1,1\right]$,而當$\left|x\right|>10\pi$時線性函數$\displaystyle y=\frac{x}{10\pi}$的取值便不落在$\left[-1,1\right]$。而$0$至$10\pi$對正弦函數經過五個週期,除了在第一個週期中僅有一個交點外,每個週期中經歷過上升與下降會有兩個交點,因此在$x>0$處有$9$個交點,故共計有$9+9+1=19$個交點,應選(3)
  3. 多選題(佔$30$分)
  4. 說明:第$6$至第$11$題,每題的五個選項各自獨立,其中至少有一個選項是正確的,選出正確選項劃記在答案卡之「解答欄」。每題皆不倒扣,五個選項全部答對者得$5$分,只錯一個選項可得$2.5$分,錯兩個或兩個以上選項不給分。
    1. 若$\Gamma=\left\{z|z為複數且\left|z-1\right|=1\right\}$,則下列哪些點會落在圖形$\Omega=\left\{w|w=iz,z\in\Gamma\right\}$上?
      1. $2i$
      2. $-2i$
      3. $1+i$
      4. $1-i$
      5. $-1+i$
    2. 訣竅直接檢查有那些點會落在$\Gamma$中,那麼乘以$i$後就會落在$\Omega$,反之不落在$\Gamma$中的點乘以$i$亦不會落在$\Omega$中。
      解法
      1. 因為$2\in\Gamma$,故$2i\in\Omega$。
      2. 因為$-2\notin\Gamma$,所以$-2i\notin\Omega$。
      3. 因為$1-i\in\Gamma$,所以$1+i\in\Omega$。
      4. 因為$-1-i\notin\Gamma$,所以$1-i\notin\Gamma$。
      5. 因為$1+i\in\Gamma$,所以$-1+i\in\Omega$。
      由以上可知應選(1)(3)(5)。

    3. 坐標平面上兩相異兩點$P$、$Q$,其中$P$點坐標為$\left(s,t\right)$。已知線段$PQ$的中垂線$L$的方程式為$3x-4y=0$,試問下列哪些選項是正確的?
      1. 向量$\overset{\rightharpoonup}{PQ}$與向量$\left(3,-4\right)$平行
      2. 線段$\overline{PQ}$的長度等於$\displaystyle\frac{\left|6s-8t\right|}{5}$
      3. $Q$點坐標為$\left(t,s\right)$
      4. 過$Q$點與直線$L$平行之直線必過點$\left(-s,-t\right)$
      5. 以$O$表示原點,則向量$\overset{\rightharpoonup}{OP}+\overset{\rightharpoonup}{OQ}$與向量$\overset{\rightharpoonup}{PQ}$的內積必為$0$
    4. 訣竅由中垂線導致的垂直與平分的特性求解。
      解法
      1. 按題意可知$\overset{\rightharpoonup}{PQ}$為$L$的法向量,又由方程本身可知$\left(3,-4\right)$亦為$L$的法向量,從而$\overset{\rightharpoonup}{PQ}$與向量$\left(3,-4\right)$平行,本選項正確。
      2. 線段的長度$\overline{PQ}$為$P$到$L$的距離的兩倍,亦即所求為

        $\displaystyle2\times\frac{\left|3\cdot s-4\cdot t\right|}{5}=\frac{\left|6s-8t\right|}{5}$

        故本選項正確。
      3. 假若$Q$之座標為$\left(t,s\right)$,那麼$\overset{\rightharpoonup}{PQ}=\left(t-s,s-t\right)=\left(t-s\right)\left(1,-1\right)$不與$\left(3,-4\right)$平行,與選項(1)衝突。因此$Q$點坐標不為$\left(t,s\right)$,本選項錯誤。
      4. 連$\overline{PQ}$交$L$的交點為$M$,記$\left(-s,-t\right)$為$K$而原點為$O$,那麼可以注意到$O$與$K$皆在$L$上。現在考慮$\Delta PQK$,$O$與$M$分別為$\overline{PK}$與$\overline{PQ}$的中點,故$\overline{QK}$與$L$平行,因此過$L$點與直線$L$平行的直線有通過$\left(-s,-t\right)$,因此本選項正確。
      5. 利用對稱性可知$\overset{\rightharpoonup}{OP}+\overset{\rightharpoonup}{OQ}$為$L$的方向向量,因此與向量$\overset{\rightharpoonup}{PQ}$垂直,從而內積為零,本選項正確。
      由以上的分析可知應選(1)(2)(4)(5)。

    5. 下列哪些選項中的矩陣經過一系列的列運算可以化成$\begin{pmatrix}1&2&3&7\\0&1&1&2\\0&0&1&1\end{pmatrix}$?
      1. $\begin{pmatrix}1&2&3&7\\0&1&1&2\\0&2&3&5\end{pmatrix}$
      2. $\begin{pmatrix}-1&3&-1&0\\-1&1&1&0\\3&1&-7&0\end{pmatrix}$
      3. $\begin{pmatrix}1&1&2&5\\1&-1&1&2\\1&1&2&5\end{pmatrix}$
      4. $\begin{pmatrix}2&1&3&6\\-1&1&1&0\\-2&2&2&1\end{pmatrix}$
      5. $\begin{pmatrix}1&3&2&7\\0&1&1&2\\0&1&0&1\end{pmatrix}$
    6. 訣竅直接進行列運算即可發現能否做到。
      解法
      1. 將第二列乘以$\left(-2\right)$加至第三列可得題幹所述的矩陣。
      2. 由於第四行(column)恆為零,因此使用任何列運算皆恆為零,故不可能使之成為題幹所述之矩陣。
      3. 由於第一列與第三列相同,因此使用列運算將使其某列恆為零,則此矩陣所對應的聯立方程組的解集合與題幹所述之矩陣不相同。
      4. 首先第二列乘以$1$加至第一列可得

        $\begin{pmatrix}1&2&4&6\\-1&1&1&0\\-2&2&2&1\end{pmatrix}$

        接著第一列乘以$1$加至第二列並且乘以$2$加至第三列可得

        $\begin{pmatrix}1&2&4&6\\0&3&5&6\\0&6&10&13\end{pmatrix}$

        將第二列乘以$-2$加至第三列可得

        $\begin{pmatrix}1&2&4&6\\0&3&5&6\\0&0&0&1\end{pmatrix}$

        若表為聯立方程組則為無解,故與題幹所述之矩陣不可能相同。
      5. 本選項所述之矩陣若寫為聯立方程組所得的唯一解為$\left(x,y,z\right)=\left(2,1,1\right)$,此與題幹所述的矩陣相同,故可經過一系列的列運算化成題幹的矩陣。
      由以上的討論可知應選(1)(5)。

    7. 坐標空間中,在$xy$平面上置有三個半徑為$1$的球兩兩相切,設其球心分別為$A,B,C$。今將第四個半徑為$1$的球置於這三個球的上方,且與這三個球都相切,並保持穩定。設第四個球的球心為$P$,試問下列哪些選項是正確的?
      1. 點$A,B,C$所在的平面和$xy$平面平行
      2. 三角形$ABC$是一個正三角形
      3. 三角形$PAB$有一邊長為$\sqrt{2}$
      4. 點$P$到直線$AB$的距離為$\sqrt{3}$
      5. 點$P$到$xy$平面的距離為$1+\sqrt{3}$
    8. 訣竅本問題之球大小相同,因此球體相切時心連線段長相等,容易形成正三角形。
      解法
      1. 點$A,B,C$所在的平面即在$xy$平面上方單位$1$之處,故與$xy$平面平行,本選項正確。
      2. 由於三顆球兩兩相切,因此$\overline{AB}=\overline{BC}=\overline{CA}=2$,故三角形$ABC$是正三角形,本選項正確。
      3. 由於第四顆球與與三顆球仍舊相切,因此三角形$PAB$每個邊長皆為$2$,故本選項錯誤。
      4. 由於$\Delta PAB$為正三角形,其邊長為$2$,故$P$到$\overline{AB}$的距離即為以$\overline{AB}$為底的高,按畢氏定理可求出高為$\sqrt{3}$,因此本選項正確。
      5. 由於$P-ABC$形成正四面體,而$P$至平面$ABC$的距離可沿用畢氏定理求得$\displaystyle\sqrt{\sqrt{3}^2-\left(\frac{\sqrt{3}}{3}\right)^2}=\frac{2\sqrt{6}}{3}$,故$P$到$xy$平面的距離為$\displaystyle1+\frac{2\sqrt{6}}{3}$,因此本選項錯誤。
      由以上可知應選(1)(2)(4)。

    9. 設$a$為大於$1$的實數,考慮函數$f\left(x\right)=a^x$與$g\left(x\right)=\log_ax$,試問下列哪些選項是正確的?
      1. 若$f\left(3\right)=6$,則$g\left(36\right)=6$
      2. $\displaystyle\frac{f\left(238\right)}{f\left(219\right)}=\frac{f\left(38\right)}{f\left(19\right)}$
      3. $g\left(238\right)-g\left(219\right)=g\left(38\right)-g\left(19\right)$
      4. 若$P,Q$為$y=g\left(x\right)$的圖形上兩相異點,則直線$PQ$之斜率必為正數
      5. 若直線$y=5x$與$y=f\left(x\right)$的圖形有兩個交點,則直線$\displaystyle y=\frac{1}{5}x$與$y=g\left(x\right)$的圖形也有兩個交點
    10. 訣竅使用指數率與對數律進行計算並注意到指數函數與對數函數的對稱性。
      解法
      1. 若$f\left(3\right)=6$,則$a^3=6$,因此$g\left(36\right)=\log_a36=2\log_a6=2\log_aa^3=2\times3=6$,故本選項正確。
      2. 直接計算可知

        $\displaystyle\frac{f\left(238\right)}{f\left(219\right)}=\frac{a^{238}}{a^{219}}=a^{19}=\frac{a^{38}}{a^{19}}=\frac{f\left(38\right)}{f\left(19\right)}$

        故本選項正確。
      3. 直接計算如下

        $\displaystyle g\left(238\right)-g\left(219\right)=\log_a238-\log_a219=\log_a\frac{238}{219}\neq\log_a\frac{38}{19}=\log_a38-\log_a19=g\left(38\right)-g\left(19\right)$

        故本選項錯誤。
      4. 由於$a>1$,故任取$y=g\left(x\right)$上的兩點$\left(x_1,\log_ax_1\right)$與$\left(x_2,\log_ax_2\right)$,可以知道當$x_1>x_2$時有$\log_ax_1>\log_ax_2$,從而兩點形成的直線斜率為正,即本選項正確。
      5. 若直線$y=5x$與$y=f\left(x\right)$的圖形有兩個交點$\left(x_1,y_1\right)$與$\left(x_2,y_2\right)$,那麼由對稱於$y=x$可知直線$\displaystyle y=\frac{1}{5}x$與$y=g\left(x\right)$相交於$\left(y_1,x_1\right)$與$\left(y_2,x_2\right)$,因此本選項正確。
      由以上分析可知應選(1)(2)(4)(5)。

    11. 設$f\left(x\right)$為一實係數三次多項式且其最高次項係數為$1$,已知$f\left(1\right)=1$,$f\left(2\right)=2$,$f\left(5\right)=5$,則$f\left(x\right)=0$在下列哪些區間必定有實根?
      1. $\left(-\infty,0\right)$
      2. $\left(0,1\right)$
      3. $\left(1,2\right)$
      4. $\left(2,5\right)$
      5. $\left(5,\infty\right)$
    12. 訣竅運用餘式定理的概念進行觀察可推測出$f$的長相。
      解法按題意可知$f\left(x\right)=\left(x-1\right)\left(x-2\right)\left(x-5\right)+x$。因為$f\left(0\right)=-10$、$f\left(1\right)=1$、$f\left(2\right)=2$、$f\left(3\right)=-1$、$f\left(5\right)=5$,故在$\left(0,1\right)$、$\left(2,3\right)$以及$\left(3,5\right)$之間各有一個根。又由代數基本定理可知$f$恰有三根,因此應選(2)(4)。

第二部分:選填題(佔$45$分)
說明:
  1. 第$A$至$I$題,將答案劃記在答案卡之「解答欄」所標示的列號(12-41)處。
  2. 每題完全答對給$5$分,答錯不倒扣,未完全答對不給分。
  1. 設實數$x$滿足$0<x<1$,且$\log_x4-\log_2x=1$,則$\displaystyle x=\underline{ \frac{⑫}{⑬} }$。(化成最簡分數)
  2. 訣竅運用對數律改寫為二次方程後解得$\log_2x$,再由$x$的範圍捨棄不合的答案。
    解法由對數律可知$\displaystyle\log_x4=\frac{1}{\log_4x}=\frac{2}{\log_2x}$,故方程可寫為

    $\displaystyle\frac{2}{\log_2x}-\log_2x=1$

    同乘以$\log_2x$可得

    $\left(\log_2x\right)^2+\log_2x-2=0$

    因此解得$\log_2x=-2$或$\log_2x=1$,因此$\displaystyle x=\frac{1}{4}$或$x=2$,而後者不合(因為題幹表示$x$介於$0$與$1$之間),故填入$⑫=1$、$⑬=4$。

  3. 在坐標平面上的$\Delta ABC$中,$P$為$\overline{BC}$邊之中點,$Q$在$\overline{AC}$邊上且$\overline{AQ}=2\overline{QC}$。已知$\overset{\rightharpoonup}{PA}=\left(4,3\right)$,$\overset{\rightharpoonup}{PQ}=\left(1,5\right)$,則$\overset{\rightharpoonup}{BC}=\left(\underline{⑭⑮},\underline{⑯⑰}\right)$。
  4. 訣竅依照條件使用分點公式求出各個座標所形成的方程式後加以求解。
    解法首先由於$\overline{AQ}=2\overline{QC}$,因此$\overline{AQ}:\overline{QC}=2:1$,故由分點公式可知

    $\displaystyle\overset{\rightharpoonup}{PQ}=\frac{1}{3}\overset{\rightharpoonup}{PA}+\frac{2}{3}\overset{\rightharpoonup}{PC}$

    因此可得

    $\displaystyle\overset{\rightharpoonup}{PC}=\frac{3}{2}\overset{\rightharpoonup}{PQ}-\frac{1}{2}\overset{\rightharpoonup}{PA}=\frac{3}{2}\left(1,5\right)-\frac{1}{2}\left(4,3\right)=\left(\frac{-1}{2},6\right)$

    進而有

    $\overset{\rightharpoonup}{BC}=2\overset{\rightharpoonup}{PC}=\left(-1,12\right)$

    因此填入$⑭=-$、$⑮=1$、$⑯=1$、$⑰=2$、

  5. 在某項才藝競賽中,為了避免評審個人主觀影響參賽者成績太大,主辦單位規定:先將$15$位評審給同一位參賽者的成績求得算術平均數,再將與平均數相差超過$15$分的評審成績剔除後重新計算平均值做為此參賽者的比賽成績。現在有一位參賽者所獲$15$位評審的平均成績為$76$分,其中有三位評審給的成績$92$、$45$、$55$應剔除,則這個參賽者的比賽成績為 ⑱⑲ 分。
  6. 訣竅按其敘述進行計算即可。
    解法其成績應計算如下

    $\displaystyle\frac{15\cdot76-92-45-55}{12}=\frac{948}{12}=79$


  7. 某巨蛋球場$E$區共有$25$排座位,此區每一排都比其前一排多$2$個座位。小明坐在正中間那一排(即第$13$排),發現此排共有$64$個座位,則此球場$E$區共有 ⑳㉑㉒㉓ 個座位。
  8. 訣竅在等差數列中,中位數乘以項數即為全體之和。
    解法依據訣竅可知所求為$64\cdot25=1600$,因此填入$⑳=1$、$㉑=6$、$㉒=0$、$㉓=0$

  9. 設$P,A,B$為坐標平面上以原點為圓心的單位圓上三點,其中$P$點坐標為$\left(1,0\right)$,$A$點坐標為$\displaystyle\left(\frac{-12}{13},\frac{5}{13}\right)$,且$\angle APB$為直角,則$B$點坐標為$\displaystyle\left(\underline{\frac{㉔㉕}{㉖㉗}},\underline{\frac{㉘㉙}{㉚㉛}}\right)$。(化成最簡分數)
  10. 訣竅由於$B$落於單位圓上會滿足特定的方程式,再由於$\angle APB$為直角,可以藉由內積為零求解。
    解法一由於$\angle APB$為直角,因此$\overline{AB}$為直徑,由因$A$與$B$落於單位圓上,因此$B$座標為$\displaystyle\left(\frac{12}{13},\frac{-5}{13}\right)$,因此填入$㉔=1$、$㉕=2$、$㉖=1$、$㉗=3$、$㉘=-$、$㉙=5$、$㉚=1$、$㉛=3$。
    解法二設$B$之座標為$\left(a,b\right)$。由於$B$落在單位圓上,因此$a^2+b^2=1$。又因$\angle APB$為直角,因此$\overset{\rightharpoonup}{PA}\cdot\overset{\rightharpoonup}{PB}=0$,即

    $\displaystyle\left(-\frac{25}{13},\frac{5}{13}\right)\cdot\left(a-1,b\right)=\overset{\rightharpoonup}{PA}\cdot\overset{\rightharpoonup}{PB}=0$

    因此有$-5\left(a-1\right)+b=0$,即$b=5\left(a-1\right)$,藉由代入消去$b$可得$a^2+25\left(a-1\right)^2=1$,即有$13a^2-25a+12=0$,從而解得$a=1$或$\displaystyle a=\frac{12}{13}$,進而有$b=0$、$\displaystyle b=\frac{-5}{13}$。由於當$a=1$時與$P$點重合,故不合理;因此$B$點坐標為$\displaystyle\left(\frac{12}{13},\frac{-5}{12}\right)$,填入$㉔=1$、$㉕=2$、$㉖=1$、$㉗=3$、$㉘=-$、$㉙=5$、$㉚=1$、$㉛=3$。

  11. 某公司生產多種款式的「阿民」公仔,各種款式只是球帽、球衣或球鞋顏色不同。其中球帽共有黑、灰、紅、藍四種顏色,球衣有白、綠、藍三種顏色,而球鞋有黑、白、灰三種顏色。公司決定紅色的球帽不搭配灰色的鞋子,而白色的球衣則必須搭配藍色的帽子,至於其他顏色間的搭配就沒有限制。在這些配色的要求之下,最多可有 ㉜㉝ 種不同款式的「阿民」公仔。
  12. 訣竅按條件進行增添與刪去即可;亦可直接條列如下。
    解法一所有可能搭配有$4\times3\times3=36$種搭配,但由於紅色球帽不能搭配灰色的鞋子,故應刪去$1\times3\times1=3$種,又白色球衣必須搭配藍色的帽子,故應刪去$3\times1\times3=9$種。又其中有重複刪去的情形,故增添$1$種(紅帽、白衣與灰鞋),因此最多共有$36-3-9+1=25$種公仔,劃入$㉜=2$、$㉝=5$。
    解法二直接條列如下
    • 球帽:黑/灰,球衣:綠/藍,球鞋:黑/白/灰,共計有$2\times2\times3=12$種
    • 球帽:紅,球衣:綠/藍,球鞋:黑/白,共計有$1\times2\times2=4$種
    • 球帽:藍,球衣:白/綠/藍,球鞋:黑/白/灰,共計有$1\times3\times3=9$種
    因此有$12+4+9=25$種,劃入$㉜=2$、$㉝=5$。

  13. 摸彩箱裝有若干編號為$1,2,\cdots,10$的彩球,其中各種編號的彩球數目可能不同。今從中隨機摸取一球,依據所取球的號數給予若干報酬。現有甲、乙兩案:甲案為當摸得彩球的號數為$k$時,其所獲報酬同為$k$;乙案為當摸得彩球的號數為$k$時,其所獲報酬為$11-k$($k =1,2,\cdots,10$)。已知依甲案每摸取一球的期望值為$\displaystyle\frac{67}{14}$,則依乙案每摸取一球的期望值為$\displaystyle\underline{ \frac{㉞㉟}{㊱㊲} }$。(化成最簡分數)
  14. 訣竅按期望值的線性特性求解。
    解法由於甲之平均每次進行所獲得的報酬為$\displaystyle\frac{67}{14}$,又因乙之報酬每次平均金額應與甲案之報酬平均金額總和為$11$,故乙案的平均每次進行所獲得的報酬為$\displaystyle11-\frac{67}{14}=\frac{87}{14}$,因此$㉞=8$、$㉟=7$、$㊱=1$、$㊲=4$。

  15. 坐標平面上有一以點$V\left(0,3\right)$為頂點、$F\left(0,6\right)$為焦點的拋物線。設$P\left(a,b\right)$為此拋物線上一點,$Q\left(a,0\right)$為$P$在$x$軸上的投影,滿足$\angle FPQ=60^\circ$,則$b=\underline{ ㊳㊴ }$。
  16. 訣竅留意拋物線的定義:拋物線上的點到準線與到焦點距離相同,再者思考$60^\circ$的條件可以如何使用。
    解法由於$\overline{VF}=3$,因此焦距為$3$,從而可知$y=0$處為準線,故$b=\overline{PQ}=\overline{PF}$。又因$\angle FPQ=60^\circ$,因此$\Delta FPQ$為正三角形,故$\overline{PQ}$的中垂線$\displaystyle y=\frac{b}{2}$通過$F$,因此$b=12$,填入$㊳=1$、$㊴=2$。

  17. 在$\Delta ABC$中,$M$為$\overline{BC}$邊之中點,若$\overline{AB}=3$,$\overline{AC}=5$,且$\angle BAC=120^\circ$,則$\tan\angle BAM=\underline{ ㊵\sqrt{㊶} }$。(化成最簡根式)
  18. 訣竅運用三角函數詳細求出各線段之長度後進而求出所求角度之正切值。
    解法利用餘弦定理求出$\overline{BC}$如下

    $\displaystyle\overline{BC}^2=\overline{AB}^2+\overline{AC}^2-2\overline{AB}\cdot\overline{AC}\cos\angle BAC=3^2+5^2-2\cdot3\cdot5\cdot\left(-\frac{1}{2}\right)=49$

    因此$\overline{BC}=7$。運用中線公式可知

    $\displaystyle\overline{AM}=\frac{\sqrt{2\left(\overline{AB}^2+\overline{AC}^2\right)-\overline{BC}^2}}{2}\frac{\sqrt{2\left(3^2+5^2\right)-7^2}}{2}=\frac{\sqrt{19}}{2}$

    對$\Delta BAM$使用餘弦定理有

    $\displaystyle\left(\frac{7}{2}\right)^2=3^2+\left(\frac{\sqrt{19}}{2}\right)^2-2\cdot3\cdot\frac{\sqrt{19}}{2}\cos\angle BAM$

    因此有

    $\displaystyle\cos\angle BAM=\frac{1}{2\sqrt{19}}$

    因此$\displaystyle\tan\angle BAM=\sqrt{75}=5\sqrt{3}$,故填入$㊵=5$、$㊶=3$。

參考公式及可能用到的數值

  1. 一元二次方程式$ax^2+bx+c=0$的公式解:$\displaystyle x=\frac{-b\pm\sqrt{b^2-4ac}}{2a}$
  2. 平面上兩點$P_1\left(x_1,y_1\right)$,$P_2\left(x_2,y_2\right)$間的距離$\overline{P_1P_2}=\sqrt{\left(x_2-x_1\right)^2+\left(y_2-y_1\right)^2}$
  3. 通過$\left(x_1,y_1\right)$與$\left(x_2,y_2\right)$的直線斜率$\displaystyle m=\frac{y_2-y_1}{x_2-x_1}$,$x_2\neq x_1$
  4. 等比級數$\left\langle ar^{k-1}\right\rangle$的前$n$項之和$\displaystyle S_n=\frac{a\cdot\left(1-r^n\right)}{1-r}$,$r\neq1$。
  5. 三角函數的和角公式:$\sin\left(A+B\right)=\sin A\cos B+\sin B\cos A$
              $\displaystyle\tan\left(\theta_1+\theta_2\right)=\frac{\tan\theta_1+\tan\theta_2}{1-\tan\theta_1\tan\theta_2}$
  6. $\Delta ABC$的正弦定理:$\displaystyle\frac{\sin A}{a}=\frac{\sin B}{b}=\frac{\sin C}{c}$
    $\Delta ABC$的餘弦定理:$c^2=a^2+b^2-2ab\cos C$
  7. 棣美弗定理:設$z=r\left(\cos\theta+i\sin\theta\right)$,則$z^n=r^n\left(\cos n\theta+i\sin n\theta\right)$,$n$為一正整數
  8. 算術平均數: $\displaystyle M\left(={\bar X}\right)=\frac{1}{n}\left(x_1+x_2+\cdots+x_n\right)=\frac{1}{n}\sum_{i=1}^{n}x_i$
    (樣本)標準差: $\displaystyle S=\sqrt{\frac{1}{n-1}\sum_{i=1}^{n}\left(x_i-{\bar X}\right)^2}=\sqrt{\frac{1}{n-1}\left(\sum_{i=1}^{n}x_i^2-\bar{X}^2\right)}$
  9. 參考數值:$\sqrt{2}\approx1.414$; $\sqrt{3}\approx1.732$; $\sqrt{5}\approx2.236$; $\sqrt{6}\approx2.449$; $\pi\approx3.142$
  10. 對數值:$\log_{10}2\approx0.3010$,$\log_{10}3\approx0.4771$,$\log_{10}5\approx0.6990$,$\log_{10}7\approx0.8451$

沒有留言:

張貼留言